0
$\begingroup$

Possible Duplicate:
Differentiability of Norms

How would you find at which point the $L_p$ norm, $$|x|_p = (|x_1|^p + \dots + |x_n|^p)^{\frac{1}{p}}$$ is differentiable?

  • 1
    Probably duplicated.2012-05-15
  • 2
    I've answered this here: http://math.stackexchange.com/questions/144449/differentiability-of-norms/144477#1444772012-05-15

0 Answers 0